Physics Technology Update (4th Edition)

Published by Pearson
ISBN 10: 0-32190-308-0
ISBN 13: 978-0-32190-308-2

Chapter 6 - Applications of Newton's Laws - Problems and Conceptual Exercises - Page 186: 107

Answer

(a) $10m/s^2$ (b) $47^{\circ}$ (c) Please see the work below.

Work Step by Step

(a) The required centripetal acceleration can be determined as follows: $a_c=\frac{v^2}{r}$ We plug in the known values to obtain: $a_c=\frac{[(25mi/h)(\frac{1h}{3600s})(\frac{1m}{6.2137\times 10^{-4}mi})]^2}{12m}$ $a_c=10.4m/s^2\approx10m/s^2$ (b) We can find the required angle as follows: The x-component of the tension is given as $Tsin\theta=F_c$ $Tsin\theta=ma_c$.....eq(1) and the Y-component of the tension is given as $Tcos\theta=mg$...eq(2) Dividing eq(1) by eq(2), we obtain: $\frac{Tsin\theta}{Tcos\theta}=\frac{ma_c}{mg}$ $tan\theta=\frac{a_c}{g}$ $\theta=tan^{-1}\frac{a_c}{g}$ We plug in the known values to obtain: $\theta=tan^{-1}(\frac{10.4m/s^2}{9.8m/s^2})$ $\theta=47^{\circ}$ (c) We know that the weight and the centripetal force both depend on the mass but the mass drops from the equation for the angle $\theta$, and thus the angle $\theta$ is independent of the weight of the rider.
Update this answer!

You can help us out by revising, improving and updating this answer.

Update this answer

After you claim an answer you’ll have 24 hours to send in a draft. An editor will review the submission and either publish your submission or provide feedback.